2
$\begingroup$

Let $\Omega$ be a bounded domain in $\mathbb R^n $ with smooth boundary.

I was wondering if there exist any known conditions on $\Omega$ such that the 1st Dirichlet eigenvalue of the (positive) Laplacian is below the $2$nd Neumann eigenvalue, i.e.

$$\lambda_1^{\operatorname{Neumann}}(\Delta) \le \lambda_1^{\operatorname{Dirichlet}}(\Delta)$$ is clear but when do we also have

$$\lambda_1^{\operatorname{Dirichlet}}(\Delta) \le \lambda_2^{\operatorname{Neumann}}(\Delta)? $$

$\endgroup$
1
  • 6
    $\begingroup$ You have always for $n \ge 2$ that $\lambda_{n+1}^{Neumann} <\lambda_n^{Dirichlet}$. The easiest prrof is due to Filonov (st. Petersburgh Math. J. 2005). $\endgroup$ Mar 19, 2020 at 18:10

0

Your Answer

By clicking “Post Your Answer”, you agree to our terms of service and acknowledge that you have read and understand our privacy policy and code of conduct.